Acute cardio Flashcards
- A 62-year-old man presents to the emergency
department (ED) with the chief concern of chest
pain that woke him from sleep and radiates to
his jaw. An electrocardiogram (ECG) reveals
ST-segment depression in leads II, III, and aVF.
His blood pressure is 112/62 mm Hg and heart rate
is 60 beats/minute. Cardiac enzymes have been
obtained, and the first troponin result was slightly
positive. Preparations are under way to take the
patient to the cardiac catheterization laboratory
for evaluation. Which medication regimen is most
appropriate for this patient at this time?
A. Aspirin 325 mg, clopidogrel 600-mg loading
dose (LD), and unfractionated heparin
(UFH) infusion 80-unit/kg bolus, followed
by 18 units/kg/hour and metoprolol 5 mg
intravenously.
B. Aspirin 81 mg; prasugrel 60-mg LD; UFH
infusion 60-unit/kg bolus, followed by 12
units/kg/hour; and intravenous enalaprilat.
C. Aspirin 325 mg, ticagrelor 180-mg LD, and
UFH infusion 60-unit/kg bolus, followed by
12 units/kg/hour.
D. Aspirin 81 mg, prasugrel 60-mg LD, nitroglycerin
infusion at 10 mcg/minute, and
bivalirudin 0.75-mg/kg bolus and 1.75-mg/kg/
hour infusion.
- Answer: C
This patient’s chest pain, ST-segment depression on
ECG, and positive biomarkers for myocardial necrosis
suggest NSTE-ACS. Because of his presentation
characteristics, he is at a high enough risk to warrant
cardiac catheterization (invasive strategy). This invasive
strategy is used to determine whether occluded or
partly occluded epicardial arteries exist, which ones
can be intervened on, and whether to do PCI (percutaneous
transluminal coronary angioplasty with or without
stenting). Initial therapy for ACS usually consists of
morphine, oxygen, nitroglycerin, and aspirin, but only
aspirin has been shown to reduce mortality from these
initial treatments. Aspirin should be given as soon
as possible after hospital presentation and continued
indefinitely, if tolerated. According to clinical trials,
guidelines, and experience, an initial dose of 162–325
mg is recommended (Answers B and D are not the
best choices of dosing for an acute episode). Aspirin,
together with a P2Y12 receptor antagonist, is indicated
for an early invasive strategy in the management of UA/
NSTEMI, improving outcomes. The 2014 NSTE-ACS
guidelines give a class I recommendation for clopidogrel,
ticagrelor, and prasugrel in ACS for patients
undergoing PCI. The choice of which P2Y12 receptor
antagonist to use in the ACS setting depends on patient
presentation, contraindications, and whether PCI is
involved; in this case, any of the three P2Y12 antagonists
would be appropriate. The anticoagulation strategy
treatment for ACS generally includes one anticoagulant
(UFH, low-molecular-weight heparin, fondaparinux, or
bivalirudin). When UFH is chosen as an anticoagulant
strategy, the dose used for ACS is a 60-unit/kg bolus
and a 12-unit/kg/hour infusion (Answer A is incorrect).
Regarding dosing, bivalirudin (Answer D) would be an
appropriate anticoagulation strategy; however, the initial
aspirin dose should be higher, and a nitroglycerin
drip would not be the best choice, given his right-sided
MI (low blood pressure, low heart rate). Answer A is
reasonable in patients without contraindications when
hypertension or ongoing ischemia is a concern; however,
initiating oral therapy within 24 hours is preferred
in most patients as long as they have no signs of HF,
evidence of low output state, increased risk of cardiogenic
shock, or other contraindications to β-blockade.
β-Blockers should initially be avoided in this patient,
given his blood pressure and decreased baseline heart
rate. An intravenous ACE inhibitor (Answer B) should
not be given to patients within the first 24 hours of ACS
because of the increased risk of hypotension. Answer C
includes DAPT and an appropriate anticoagulant dose
(Answer C is correct).
- An 81-year-old African American man (weight 90
kg) presents to the ED with chest pressure (10/10
on a pain scale). His ECG reveals ST-segment
depression in the inferior leads. His medical history
is significant for hypertension and chronic
kidney disease. Pertinent laboratory results are troponin
5.8 ng/L, serum creatinine (SCr) 3.7 mg/dL,
and estimated creatinine clearance (eCrCl) 20 mL/
minute. The patient has been given aspirin 325
mg single dose; a nitroglycerin drip, initiated at 5
mcg/minute, will be titrated to chest pain relief and
blood pressure. The patient consents for cardiac
catheterization after adequate hydration. Which
anticoagulation strategy is most appropriate to initiate
in this patient?
A. Intravenous heparin 4000-unit intravenous
bolus, followed by a 1000-unit/hour continuous
infusion.
B. Enoxaparin 90 mg subcutaneously every 12
hours.
C. Fondaparinux 2.5 mg subcutaneously daily.
D. Bivalirudin 67.5-mg bolus, followed by a 157-
mg/hour infusion.
- Answer: A
The NSTE-ACS guidelines recommend the use of one
anticoagulant during an acute event. Enoxaparin, UFH,
and bivalirudin are all recommended as class I agents
for the invasive management of NSTE-ACS. However,
fondaparinux (Answer C) is not optimal because of the
increased risk of catheter-related thrombosis associated
with its use in the catheterization laboratory. The
NSTE-ACS guidelines advise the use of an additional
anticoagulant with class IIa activity (heparin or bivalirudin)
if fondaparinux was an initial anticoagulant
when the patient underwent intervention, whereas the
PCI guidelines give fondaparinux a class III or harmful
recommendation. Of the remaining three options, UFH
(Answer A) is preferred because of its dosage and rapid
clearance regardless of renal function. The UFH bolus
should be limited to 4000 units, and the initial infusion
should be limited to 1000 units/hour. Both enoxaparin
(Answer B) and bivalirudin (Answer D) would be
appropriate but would need to be dose adjusted, given
this patient’s CrCl of less than 30 mL/minute. However,
the doses in Answers B and D would be appropriate for
patients with a normal CrCl.
- A 56-year-old man presents to the hospital with
the chief concern of chest pain that was unrelieved
at home with sublingual nitroglycerin. His ECG
reveals ST-segment depression and T-wave inversion.
Cardiac markers show an elevated troponin I.
The cardiologist has requested that the patient go
to the cardiac catheterization laboratory for further
evaluation. The patient has a history of coronary
artery disease (CAD) and had a myocardial infarction
(MI) about 6 months ago. During his previous
hospitalization, he was confirmed to have developed
heparin-induced thrombocytopenia (HIT)
after his platelet count (Plt) dropped to 40,000/
mm3 and he had a positive ELISA (enzyme-linked
immunosorbent assay) upon serologic testing after
his previous catheterization. Given this patient’s
diagnosis and history, which treatment regimen
would be most appropriate during his cardiac
catheterization?
A. Tirofiban.
B. Bivalirudin.
C. Enoxaparin.
D. Tenecteplase
- Answer: B
An anticoagulant is required for PCI. Options include
UFH, bivalirudin, and enoxaparin. Because this patient
had a significantly low Plt with his most recent heparin
exposure and was confirmed to have HIT, using any
of the GP IIb/IIIa inhibitors (Answers A and C) would
be unwise for ACS treatment because these agents are
usually combined with UFH. Furthermore, GP IIb/
IIIa inhibitors are antiplatelets, and the patient will still
need an additional agent with anticoagulant activity.
Thrombolytic therapy is not recommended for NSTEACS
and would be inappropriate in this patient (Answer
D is incorrect). Answer D is also incorrect because
enoxaparin carries a 10% risk of cross-reactivity if HIT
is suspected. Bivalirudin (Answer B), a direct thrombin
inhibitor, would be the treatment of choice in patients
with HIT undergoing PCI.
- A 62-year-old man presents to the ED after several
hours of chest discomfort. His ECG reveals
a 1- to 2-mm ST-segment elevation with positive
troponins. He has also had increasing shortness of
breath and lower-extremity swelling over the past
2–3 weeks. His medical history is significant for
tobacco use for 40 years, chronic obstructive pulmonary
disease, diabetes, and hypertension. His
blood pressure is 102/76 mm Hg and heart rate is
111 beats/minute. He has rales in both lungs and
2–3+ pitting edema in his extremities. His echocardiogram
reveals an ejection fraction (EF) of 25%.
After primary percutaneous coronary intervention
(PCI), he is transferred to the cardiac intensive
care unit. Which best describes the acute use of
β-blocker therapy in this patient?
A. Give 12.5 mg of oral carvedilol within the first
24 hours.
B. Give 5 mg of intravenous metoprolol at the
bedside.
C. Give 200 mg of oral metoprolol succinate at
discharge.
D. Give no β-blocker at this time.
- Answer: D
β-Blocker therapy can cause HF decompensation, particularly
when the β-blocker is titrated too quickly or
initiated in patients who are not euvolemic. Although
administering β-blockers within the first 24 hours is
beneficial in STEMI, this patient has several risk factors
that would be considered contraindications to initial
β-blockade. This patient’s clinical condition suggests
he is not euvolemic, and aggressive diuresis should be
tried before a β-blocker is initiated for him. In addition,
intravenous β-blocker therapy (Answer B) would
place him at an even greater risk of cardiogenic shock.
Answers A and C are inappropriate because the doses
are fairly aggressive for a patient with marginal blood
pressure at this time. Answer D is correct; however,
before discharge, this patient should be reevaluated for
the initiation of low-dose β-blocker therapy.
- A 60-year-old man (weight 75 kg) presents to
the ED with crushing substernal chest pain and
ST-segment elevations on ECG. He has a medical
history of diabetes and a 40 pack-year history of
smoking. He is taken immediately to the catheterization
laboratory for primary PCI, and a drugeluting
stent is placed in his left anterior descending
artery. In addition to aspirin, which regimen
would best maintain this patient’s stent patency?
A. Clopidogrel 300-mg LD, followed by 75 mg
daily for 12 months.
B. Prasugrel 60-mg LD, followed by 10 mg daily
for 12 months.
C. Ticagrelor 180-mg LD, followed by 90 mg
daily for 6 months.
D. Clopidogrel 600-mg LD, followed by 75 mg
daily for 6 months.
- Answer: B
Dual antiplatelet therapy is recommended for at least 12
months in patients presenting with ACS. Early discontinuation
of DAPT is reasonable when the risk of morbidity
exceeds the expected benefit (class IIa), as in the case of
bleeding. Answers C and D do not represent the minimum
time interval, given that the patient has no known
risk of bleeding. Prasugrel (Answer B) would be preferable
to clopidogrel (Answer A) in this scenario because
it would be faster in onset; clopidogrel would take about
6 hours for maximal platelet inhibition after a 300-mg
LD. Subgroup analysis of a Triton-TIMI 38 comparing
the effectiveness of prasugrel and clopidogrel showed the
superiority of prasugrel, especially for patients presenting
with a STEMI (Answer B is correct).
- A 60-year-old woman with New York Heart
Association (NYHA) class IV heart failure (HF)
(heart failure with reduced ejection fraction
[HFrEF]) is admitted for increased shortness of
breath and dyspnea at rest. Her extremities appear well perfused, but she has 3+ pitting edema in her
lower extremities. Her vital signs include blood
pressure 125/70 mm Hg, heart rate 92 beats/
minute, and oxygen saturation (Sao2) 89% on 100%
facemask. After initiating an intravenous diuretic,
which intravenous agent is best to rapidly treat this
patient’s pulmonary symptoms?
A. Dobutamine.
B. Milrinone.
C. Nitroglycerin.
D. Metoprolol.
- Answer: C
This patient is well perfused and can be classified in
Forrester hemodynamic subset II (warm and wet).
Because the patient has pulmonary congestion (shortness
of breath, dyspnea at rest), intravenous diuretics are
first-line therapy. Nitroglycerin (Answer C) is best in
this setting because vasodilatory agents can be used in
conjunction with intravenous diuretics to improve acute
pulmonary edema. When adjunctive therapy is needed
in addition to loop diuretics, intravenous vasodilators
should be considered over inotropic agents when blood
pressure is adequate. Dobutamine (Answer A) and milrinone
(Answer B) primarily increase CO, which is not
a problem in warm and wet exacerbations. In addition,
the adverse effects of these agents (increased mortality,
- A 75-year-old woman admitted for pneumonia
has a history of several non–ST-segment elevation
myocardial infarctions (NSTEMIs). She had an
episode of sustained ventricular tachycardia (VT)
during this hospitalization. Her corrected QT (QTc)
interval was 380 milliseconds on the telemetry.
Her left ventricular ejection fraction (LVEF) was
found to be 25%. Her serum potassium and magnesium
were 4.6 mEq/L and 2.2 mg/dL, respectively.
Which intravenous agent is most appropriate for
this patient’s ventricular arrhythmias?
A. Procainamide.
B. Metoprolol.
C. Magnesium.
D. Amiodarone.
- Answer: D
This patient has a depressed LVEF less than 40%;
therefore, her AAD therapy options are limited.
Procainamide (Answer A) is indicated only in secondary
prevention of sustained VT in patients with a normal
LVEF greater than 40%; if given to this patient, it
could worsen her HF. Metoprolol (Answer B) is indicated
for treating patients with asymptomatic nonsustained
VT and SVT associated with CAD. This patient
had an episode of sustained VT; therefore, therapy
beyond β-blockade is warranted. Her QTc interval is
not prolonged at 380 milliseconds, and her serum magnesium
concentration is within normal limits; thus, she
does not need intravenous magnesium therapy (Answer
C). Amiodarone (Answer D) is first line for patients
without contraindications because of its efficacy and
safety in patients with an LVEF less than 40%.
- A 53-year-old woman is admitted to the hospital
after the worst headache she has ever had. Her
medical history includes exertional asthma, poorly
controlled hypertension, glaucoma, and hyperlipidemia.
She is nonadherent to her medications
and has not taken her prescribed blood pressure
medications for 4 days. Vital signs include blood
pressure 220/100 mm Hg and heart rate 65 beats/
minute. She has retinal hemorrhaging on funduscopic
examination. Which is most appropriate for
this patient’s hypertensive emergency?
A. Fenoldopam 0.1 mcg/kg/minute.
B. Nicardipine 5 mg/hour.
C. Labetalol 0.5 mg/minute.
D. Enalaprilat 0.625 mg intravenously every 6
hours.
- Answer: B
This patient has target-organ damage from poorly
controlled hypertension in the form of retinal hemorrhaging.
Fenoldopam is contraindicated for treating
hypertensive emergencies in the setting of glaucoma
(Answer A is incorrect). Nicardipine is appropriate for
this patient, given the details of this case (Answer B
is correct). Although labetalol is effective for treating
hypertensive emergency, this patient has a history of
asthma and a low heart rate, making labetalol a lessthan-
ideal option for treating her symptoms (Answer C
is incorrect). The antihypertensive effects of enalaprilat
depend on a patient’s renin activity, which is unknown
in this case. Therefore, the blood pressure–reducing
effects may be more difficult to control than when using
a drug having a more consistent effect in individuals.
In addition, the bolus nature of the drug is not ideal for
tightly controlling blood pressure with no more than a
25% reduction in MAP. Continuous infusion drugs are
preferable for easier titration to effect in a hypertensive
emergency (Answer D is incorrect).
- A 52-year-old woman has a witnessed cardiac
arrest in a shopping mall and is resuscitated with
an automatic external defibrillator device. On electrophysiologic
study, she has inducible VT. Which
is most appropriate for reducing the secondary
incidence of sudden cardiac death (SCD)?
A. Propafenone.
B. Amiodarone.
C. Implantable cardioverter-defibrillator (ICD).
D. Metoprolol.
- Answer: C
The Cardiac Arrest Study Hamburg trial compared ICD
with AAD in survivors of cardiac arrest for secondary
prevention of SCD. The propafenone (Answer A) study arm was discontinued early because of its significantly
(61%) higher mortality rate compared with the ICD arm
(Answer A is incorrect). Although this trial had a small
sample size that prevented a statistically significant difference
in total mortality in ICD-treated patients versus
patients treated with either amiodarone or metoprolol,
the incidence of sudden death was significantly lower in
patients with an ICD (Answer C is correct; 33% vs. 13%;
p=0.005). The Antiarrhythmics Versus Implantable
Defibrillators trial also evaluated ICD implantation versus
AAD therapy (primarily amiodarone) in survivors
of SCD. Patients with ICDs had a significantly higher
rate of survival than did those treated with drug therapy
(89% vs. 82%; p<0.02), making Answer C preferable to
all the other options (Answers B and D are incorrect).
- The Sudden Cardiac Death in Heart Failure trial
evaluated the efficacy of amiodarone or an ICD
versus placebo in preventing all-cause mortality
in ischemic and nonischemic patients with NYHA
class II and III HF. There was a 7.2% absolute risk
reduction and a 23% relative risk reduction in allcause
mortality at 60 months with an ICD versus
placebo. Which best shows the number of patients
needed to treat with an ICD to prevent one death
versus placebo?
A. 1.
B. 4.
C. 14.
D. 43.
- Answer: C
The number needed to treat can be calculated as 1/absolute
risk reduction. Because the absolute risk reduction
in mortality at 60 months was 7.2% with ICD versus placebo,
1/0.072 would be used to calculate the number of
patients needed to treat to prevent one death during this
time. About 14 patients (Answer C) would need to be
treated with ICD to prevent one death in 60 months versus
placebo. Other calculations in this fashion, including
relative risk reduction and 100% minus the absolute or
relative risk reduction, provide no useful information
for interpreting the trial results and yield an incorrect
number of patients (Answers A, B, and D are incorrect).
Patient Cases
1. A 66-year-old woman (weight 70 kg) with a history of MI, hypertension, hyperlipidemia, and diabetes mellitus
presents with sudden-onset diaphoresis, nausea, vomiting, and dyspnea, followed by a bandlike upper
chest pain (8/10) radiating to her left arm. She had felt well until 1 month ago, when she noticed her typical
angina was occurring with less exertion. Her ECG reveals ST-segment depression in leads II, III, and aVF and
hyperdynamic T waves and positive cardiac enzymes. Blood pressure is 150/90 mm Hg, and all laboratory
results are normal; SCr is 1.2 mg/dL. Home medications are aspirin 81 mg/day, simvastatin 40 mg every night,
metoprolol 50 mg twice daily, and metformin 1 g twice daily. Which regimen is best for this patient?
A. Aspirin 325 mg, ticagrelor 180 mg one dose, and UFH 60-unit/kg bolus; then 12 units/kg/hour titrated
to 50–70 seconds with an early invasive approach.
B. Aspirin 325 mg and enoxaparin 70 mg subcutaneously twice daily with an early invasive approach.
C. An ischemia-guided strategy with tirofiban 25 mcg/kg; then 0.15 mg/kg/minute plus enoxaparin 80 mg
subcutaneously twice daily, aspirin 325 mg/day, and clopidogrel 300 mg one dose; then 75 mg once daily.
D. An ischemia-guided strategy with aspirin 325 mg and ticagrelor 180 mg one dose; plus UFH 70-unit/kg
bolus; then 15 units/kg/hour.
- Answer: A
This patient’s atypical symptoms, ST-segment depression
on ECG, and positive biomarkers for myocardial
necrosis suggest NSTE-ACS. She has at least three risk
factors for CAD, a history of CAD (prior MI), and positive
troponins, which place her at high risk of future
events. In such high-risk patients, an early invasive
strategy (as in Answers A and B) is used to determine
whether occluded or partly occluded arteries exist,
which ones can be intervened on, and whether to make
an intervention. An ischemia-guided approach, also
called “medical management,” as in Answers C and D,
would not be preferred because of this patient’s risk category
(i.e., positive troponins). Dual antiplatelet therapy
(aspirin plus a P2Y12 inhibitor) is indicated for an
early invasive strategy in managing an NSTE-ACS. In
patients undergoing PCI, clopidogrel, prasugrel, or ticagrelor
is appropriate (Answer A is correct). After an initial
bolus of 60 units/kg and an infusion of 12 units/kg/
hour (Answer A is correct; Answer D is incorrect), UFH
can be titrated to an aPTT of 50–70 seconds. Aspirin
alone without a P2Y12 agent, as in Answer B, would
not provide adequate antiplatelet therapy. Furthermore,
enoxaparin would need to be dosed with a 30-mg intravenous
bolus before initiating twice-daily subcutaneous
dosing because this patient has positive troponins.
Glycoprotein IIb/IIIa inhibitors, as in Answer C, can be
useful in high-risk patients, typically those with positive
troponins; however, their benefit has been shown
mainly when UFH, not low-molecular-weight heparin,
is given as the anticoagulant.
Patient Cases (Cont’d)
2. A 45-year-old patient underwent an elective percutaneous transluminal coronary angioplasty and drug-eluting
stent placement in the right coronary artery. Which best represents the minimum time DAPT should be continued?
A. 1 month.
B. 3 months.
C. 6 months.
D. 12 months.
- Answer: C
Of importance, this patient case occurs in the context of
elective stent placement, not after ACS. In the non-ACS
setting, the duration of DAPT is determined by the type
of stent placed (bare metal stent vs. drug-eluting stent).
After elective drug-eluting stent placement, DAPT is
recommended for at least 6 months because the risk of
in-stent thrombosis is highest during this time (Answer
C is correct). The recommendation is for at least 1
month after bare metal stent placement because endothelialization
of the stent usually occurs early, typically
within 1 month after stenting (Answer A is incorrect).
Bleeding risk may be a reason to consider earlier termination
(after at least 1 month) of DAPT after bare
metal stent placement. Although European guidelines consider 3 months of DAPT after drug-eluting stent
placement appropriate, the recommended minimum
according to the U.S. guidelines is 6 months (Answer
B is incorrect). Twelve months of therapy is recommended
after ACS (Answer D is incorrect).
- A 52-year-old man (weight 100 kg) with a history of hypertension and hypertriglyceridemia presents at a major
university teaching hospital with a cardiac catheterization laboratory. He has had 3 hours of crushing 10/10
substernal chest pain radiating to both arms that began while he was eating his lunch (seated), which is accompanied
by nausea, diaphoresis, and shortness of breath. He has never before had chest pain of this character or
intensity. He usually can walk several miles without difficulty and smokes 1.5 packs/day of cigarettes. Home
medications are lisinopril 2.5 mg/day and aspirin 81 mg daily. Current vital signs include heart rate 68 beats/
minute and blood pressure 178/94 mm Hg. His ECG reveals a 3-mm ST-segment elevation in leads V2–V4, I,
and aVL. Serum chemistry values are within normal limits. The first set of cardiac markers shows positive
troponins, 0.8 mcg/L (normal defined as less than 0.1 mcg/L). Which regimen is best for this patient’s STEMI?
A. Reperfusion with primary PCI and stenting of occluded artery, together with tirofaban 25 mcg/kg; then
0.15 mg/kg/minute, clopidogrel 300 mg one dose, and aspirin 325 mg one dose.
B. Reperfusion with a reteplase 10-unit bolus twice, 30 minutes apart, plus a UFH 60-unit/kg bolus and a
12-unit/kg/hour infusion, clopidogrel, and aspirin.
C. Reperfusion with tenecteplase 25-mg intravenous push one dose, enoxaparin 30-mg intravenous bolus
plus 100 mg subcutaneously twice daily, aspirin 325 mg one dose, ticagrelor 180 mg one dose, and bivalirudin
0.75 mg/kg followed by 1.75 mg/kg/hour.
D. Reperfusion with primary PCI with stenting, prasugrel 60 mg one dose, aspirin 325 mg one dose, and
bivalirudin 0.75 mg/kg followed by 1.75 mg/kg/hour.
- Answer: D
Because the patient presents to a hospital that can do a
primary PCI with stent implantation, this is the preferred
reperfusion strategy (Answers B and C are incorrect).
Answer A is incorrect because an anticoagulant agent
must be administered in addition to antiplatelet therapy.
Reperfusion with fibrinolytic therapy (Answers B and
C) would only be considered if PCI were expected to be
delayed by more than 120 minutes. In addition, Answer
C is incorrect because bivalirudin has not been studied
with lytic therapy. Answer D – reperfusion with primary
PCI, DAPT with aspirin and prasugrel, and dosing of
bivalirudin as an anticoagulant strategy – is correct.
- A 76-year-old male smoker (weight 61 kg) has a history of hypertension, benign prostatic hypertrophy, and
lower back pain. Three weeks ago, he began to have substernal chest pain with exertion (together with dyspnea),
which radiated to both arms and was associated with nausea and diaphoresis. These episodes have
increased in frequency to four or five times daily; they are relieved with rest. He has never had an ECG. Today,
he awoke with 7/10 chest pain and went to the ED of a rural community hospital 2 hours later. He was acutely
dyspneic and had ongoing pain. Home medications are aspirin 81 mg/day for 2 months, doxazosin 2 mg/day,
and ibuprofen 800 mg three times daily. Vital signs include heart rate 42 beats/minute (sinus bradycardia) and
blood pressure 104/48 mm Hg. Laboratory results include blood urea nitrogen (BUN) 45 mg/dL, SCr 2.5 mg/
dL, and troponin 1.5 ng/L (normal value less than 0.1 ng/L). His ECG reveals a 3-mm ST-segment elevation.
Aspirin, ticagrelor, and sublingual nitroglycerin were given in the ED. The nearest hospital with a catheterization
laboratory facility is 2½ hours away. Which regimen is best?
A. Give alteplase 15 units intravenously plus enoxaparin 30-mg intravenous bolus.
B. Use an ischemia-guided treatment strategy with UFH 4000-unit intravenous bolus, followed by 800 units
intravenously per hour.
C. Give tenecteplase 35 mg intravenously plus UFH 4000-unit intravenous bolus followed by 800 units intravenously
per hour.
D. Transfer the patient to a facility for primary PCI.
- Answer: C
Unlike the patient in case 3, this patient presents with a
STEMI to a rural community hospital where the nearest
hospital with catheterization laboratory facilities is
more than 120 minutes away (i.e., lytics are indicated).
He presents within the window for fibrinolytic therapy
consideration (less than 6 hours after chest pain
onset) and has no obvious contraindications. Because
he is still having ischemic chest pain and ST-segment
elevation, he should benefit from reperfusion therapy.
He is experiencing bradycardia, which could indicate
an occlusion above the area perfusing his sinoatrial or
atrioventricular nodes. Enoxaparin is a treatment option
for anticoagulant therapy given in conjunction with
fibrinolytics, but the patient is at higher risk of bleeding
from impaired enoxaparin clearance and needs a
dosage adjustment. Furthermore, he is older than 75,
beyond the age at which the intravenous bolus should be
given, and the alteplase dosing is incomplete (Answer
A is incorrect). Simply treating this patient conservatively
with UFH alone in the setting of ongoing chest
pain, shortness of breath, and pulmonary edema is not
optimal (Answer B is incorrect). Diagnostic catheterization
and possible PCI to determine whether an artery
can be reperfused may be desirable but is complicated
because the patient’s SCr is elevated (2.5 mg/dL), and
he is in a rural hospital, where he cannot be assessed quickly enough (within 90–120 minutes) (Answer D
is incorrect). Because of the shorter half-life and ease
of administration of tenecteplase, tenecteplase may be
preferable to alteplase. Clearance of UFH with tenecteplase
is not as altered as with enoxaparin, and it would
be a more appropriate therapy than enoxaparin in combination
with a thrombolytic (Answer C is correct).
Patient Case
Questions 5–7 pertain to the following case.
A 72-year-old man is admitted to the hospital for HF decompensation. The patient has progressively increased
dyspnea when walking (now 10 ft [3 m], previously 30 ft [6 m]) and orthopnea (now four pillows, previously two
pillows), increased bilateral lower-extremity swelling (3+), 13 kg of weight gain in the past 3 weeks, and dietary
nonadherence. He has a history of idiopathic dilated cardiomyopathy (LVEF 25%, NYHA class III), paroxysmal
AF, and hyperlipidemia. Pertinent laboratory values are as follows: BNP 2300 pg/mL (0–50 pg/mL), K+ 4.9
mEq/L, BUN 32 mg/dL, SCr 2.0 mg/dL (baseline 1.9 mg/dL), aspartate aminotransferase (AST) 40 IU/L, alanine
aminotransferase 42 IU/L, INR 1.3, aPTT 42 seconds, blood pressure 108/62 mm Hg, heart rate 82 beats/minute,
and Sao2 95%. Home medications include carvedilol 12.5 mg twice daily, lisinopril 40 mg/day, furosemide 80 mg
twice daily, spironolactone 25 mg/day, and digoxin 0.125 mg/day.
5. Which regimen is best for treating his ADHF?
A. Carvedilol 25 mg twice daily.
B. Sodium nitroprusside 0.1 mcg/kg/min IV.
C. Furosemide 120 mg intravenously twice daily.
D. Milrinone 0.5 mcg/kg/minute.
- Answer: C
This patient, who has ADHF, is receiving a β-blocker.
Although long-term β-blockers can improve HF symptoms
and reduce mortality, they can also worsen symptoms
in the short term. It is recommended to keep
the maintenance β-blocker therapy at the same or at a
slightly lower dose compared with the outpatient therapy
in patients with ADHF; increasing the β-blocker
dose before reaching euvolemia might acutely worsen
his clinical picture (Answer A is incorrect). In patients
admitted with volume overload without substantial signs
of reduced CO, it is reasonable to try intravenous loop
diuretics initially (Answer C is correct). As gut edema
increases, oral loop diuretics (notably furosemide)
become less effective because of decreased absorption.
Sodium nitroprusside could be useful if signs and
symptoms of hypoperfusion were present. Because this
patient presents mainly with warm and wet symptoms,
diuretics would be first-line (Answer B is incorrect).
Milrinone is an inotropic drug. Because of their adverse
effects, inotropes are recommended in cold and wet
exacerbations only after vasodilatory medications have
failed (Answer D is incorrect).
- After being initiated on intravenous loop diuretics with only minimal urinary output, the patient is transferred
to the coronary care unit for further management of diuretic-refractory decompensated HF. His Sao2 is now
87% on a 4-L nasal cannula, and an arterial blood gas is being obtained. His blood pressure is 110/75 mm
Hg and heart rate is 75 beats/minute. The patient’s SCr and K+ concentrations have begun to rise and are now
2.7 mg/dL and 5.4 mmol/L, respectively. In addition to a one-time dose of intravenous chlorothiazide, which
regimen is most appropriate for this patient?
A. Nitroglycerin 20 mcg/minute.
B. Sodium nitroprusside 0.3 mg/kg/minute.
C. Dobutamine 5 mcg/kg/minute.
D. Milrinone 0.5 mcg/kg/minute.
- Answer: A
Intravenous vasodilators such as nitroglycerin (Answer
A) and sodium nitroprusside (Answer B) are reasonable
if intravenous diuretics fail and the patient progresses
to acute pulmonary edema. Both agents rapidly cause
venous vasodilation and reduce pulmonary filling
pressures, which can relieve acute shortness of breath.
Answer A, nitroglycerin, is optimal for this patient,
given his declining renal function and the concern
for increased risk of thiocyanate toxicity with sodium
nitroprusside in this setting (Answer B is incorrect).
Dobutamine is typically used in states of low CO
decompensation and is counteracted by concomitant
β-blocker therapy, making it a poor choice in patients
receiving β-blockers (Answer C is incorrect). Although
milrinone is a more acceptable inotropic agent in a
patient receiving β-blockers, the dosing strategy is inappropriate as an initial dose (Answer D is incorrect).
Finally, inotropes are generally reserved for patients
when other therapies have failed.
- The patient initially responds with 2 L of urinary output overnight, and his weight decreases by 1 kg the next
day. However, by day 5, his urinary output has diminished again, and his SCr has risen to 4.3 mg/dL. He was
drowsy and confused this morning during rounds. His extremities are cool and cyanotic, blood pressure is
89/58 mm Hg, and heart rate is 98 beats/minute. It is believed that he is no longer responding to his current
regimen. A Swan-Ganz catheter is placed to determine further management. Hemodynamic values are cardiac
index 1.5 L/minute/m2, SVR 2650 dynes/second/cm5, and PCWP 30 mm Hg. Which regimen is most appropriate
for his current symptoms?
A. Milrinone 0.2 mcg/kg/minute.
B. Dobutamine 10 mcg/kg/minute.
C. Sodium nitroprusside 0.1 mcg/kg/minute.
D. Phenylephrine 20 mcg/minute.
- Answer: A
Signs of a decreased CO state in HF (e.g., increased
SCr, decreased mental status, cool extremities) suggest
a cold and wet state, and adjunctive therapy is
indicated. Positive inotropic agents such as milrinone
will increase CO to maintain perfusion to vital organs.
Milrinone will also vasodilate the peripheral vessels
to unload the heart (lower SVR). Although dobutamine
would be a potential choice in this patient, it is
not recommended in patients receiving β-blockers, and
the initial starting dose is too aggressive (Answer B is
incorrect). Although this patient has low blood pressure,
his elevated SVR suggests that he will tolerate
the vasodilatory effects of milrinone as long as it is
appropriately renally adjusted for worsening renal dysfunction
(Answer A is correct). Nitroprusside would be
relatively contraindicated in patients with an SBP less
than 100 mm Hg and is absolutely contraindicated in
patients with an SBP less than 90 mm Hg (Answer C is
incorrect). Phenylephrine has no positive beta effects;
therefore, it will not augment contractility. In addition,
it will cause vasoconstriction through alpha stimulation,
which will further increase SVR and probably
worsen CO (Answer D is incorrect). Vasoconstrictors
are reserved for patients in cardiogenic shock.
Patient Cases
Questions 8 and 9 pertain to the following case.
A 68-year-old man is admitted after an episode of syncope, with a presyncopal syndrome of seeing black spots
and dizziness before passing out. Telemetry monitor showed sustained VT for 45 seconds. His medical history
includes HF NYHA class III, LVEF 30%, two MIs, hypertension for 20 years, LV hypertrophy, DM, and diabetic
nephropathy. His medications include lisinopril 5 mg/day, furosemide 20 mg twice daily, metoprolol 25 mg twice
daily, digoxin 0.125 mg/day, glipizide 5 mg/day, atorvastatin 40 mg, and aspirin 81 mg/day. His blood pressure is
120/75 mm Hg, with heart rate 80 beats/minute, BUN 30 mg/dL, and SCr 2.2 mg/dL.
8. Which is the best therapy to initiate for conversion of his sustained VT?
A. Amiodarone 150 mg intravenously for 10 minutes, then 1 mg/minute for 6 hours, then 0.5 mg/minute.
B. Sotalol 80 mg twice daily titrated to QTc of about 450 milliseconds.
C. Dofetilide 500 mcg twice daily titrated to QTc of about 450 milliseconds.
D. Procainamide 20 mg/minute, with a maximum of 17 mg/kg.
- Answer: A
Treatment options for sustained VT depend on concomitant
disease states, particularly LVEF (40% cutoff). In
a patient with LV dysfunction, class I agents such as
procainamide are contraindicated (Answer D is incorrect).
In a patient whose CrCl is less than 60 mL/minute,
sotalol requires a considerable dose reduction to avoid
an excess risk of torsades de pointes. Sotalol is not an
effective cardioversion drug but is more useful for preventing
future episodes of arrhythmias (maintaining
sinus rhythm) once sinus rhythm is achieved (Answer
B is incorrect). Dofetilide is indicated only for AF, not
for ventricular arrhythmias; similarly, cardioversion
rates with dofetilide are low (Answer C is incorrect).
Amiodarone is first-line therapy for sustained VT in
patients with severe renal insufficiency, HF, and SHD
(Answer A is correct).
- The patient presents to the ED 3 months after amiodarone maintenance initiation (he refused ICD placement)
after a syncopal episode, during which he lost consciousness for 30 seconds, according to witnesses. He also
has rapid heart rate episodes during which he feels dizzy and lightheaded. He feels very warm all the time (he
wears shorts, even though it is winter), cannot sleep, and has lost 3 kg in weight. He received a diagnosis of
hyperthyroidism caused by amiodarone therapy. On telemetry, he has runs of nonsustained VT. Which best
predicts the duration of amiodarone-associated hyperthyroidism in this patient?
A. 12 hours.
B. 1 month.
C. 6 months.
D. 18 months.
- Answer: C
With the prolonged half-life of amiodarone and extensive
fat tissue volume of distribution, hyperthyroid
adverse effects would be expected to last for 3–5 halflives
of the drug, which is 5–8 months (Answer C is
correct; Answer A is incorrect). Although therapeutic
concentrations may decrease substantially by then,
1 month is too soon for the effects to subside (Answer
B is incorrect). Although some iodine and amiodarone
molecules will probably remain absorbed in fat stores
for years, if not for life, therapeutic concentrations
should not exist for longer than what is predicted by the
half-life (Answer D is incorrect).
- A 64-year-old woman presents to the ED with the chief concern of palpitations. Her medical history includes
hypertension controlled with a diuretic and an inferior-wall MI 6 months ago. She is pale and diaphoretic but
can respond to commands. The patient’s laboratory values are within normal limits. Her vital signs include
blood pressure 95/70 mm Hg and heart rate 145 beats/minute; telemetry shows sustained VT. Despite chronic
use of β-blocker therapy, the patient has developed sustained VT that is successfully terminated with lidocaine.
Subsequent electrophysiologic testing reveals inducible VT, and sotalol 80 mg orally twice daily is prescribed.
Two hours after the second dose, the patient’s QTc is 520 milliseconds. Which regimen change would
be most appropriate for this patient?
A. Continue sotalol at 80 mg orally twice daily.
B. Increase sotalol to 120 mg orally twice daily.
C. Discontinue sotalol and initiate dofetilide 125 mcg orally twice daily.
D. Discontinue sotalol and initiate amiodarone 400 mg orally three times daily.
- Answer: D
This patient is having QT prolongation with sotalol,
placing her at an elevated risk of developing lifethreatening
torsades de pointes. Sotalol should be discontinued
immediately (Answers A and B are incorrect).
Given the QT prolongation that occurred with
sotalol, the same will probably occur with dofetilide
(Answer C is incorrect). Amiodarone is associated with
minimal risk of torsades de pointes and therefore would
be an appropriate alternative agent to prevent ventricular
arrhythmias (Answer D is correct).
Patient Cases
11. A 68-year-old man with a history of stage 5 chronic kidney disease receiving hemodialysis, hypertension,
CAD post-MI, HFrEF, and gastroesophageal reflux disease presents with acute-onset shortness of breath and
chest pain. After his recent dialysis, he was nonadherent to medical therapy for 2 days and noticed he had
gained 2 kg in 24 hours. His baseline orthopnea worsened to sleeping sitting up in a chair for the 2 nights
before admission. He admits smoking cocaine within the past 24 hours and developed acute-onset chest tightness
with diaphoresis and nausea, and his pain was 7/10. He went to the ED, where his blood pressure was
250/120 mm Hg. He had crackles halfway up his lungs on examination, and chest radiography detected bilateral
fluffy infiltrates with prominent vessel cephalization. His ECG revealed sinus tachycardia, heart rate 122
beats/minute, and ST-segment depressions in leads 2, 3, and aVF. He was admitted for a hypertensive emergency.
Laboratory results are as follows: BUN 48 mg/dL, SCr 11.4 mg/dL, BNP 2350 pg/mL, troponin T 1.5
ng/L (less than 0.1 mcg/L), creatine kinase 227 units/L, and creatine kinase-MB 22 units/L. Which medication
is best for this patient’s hypertensive emergency?
A. Intravenous nitroglycerin 5 mcg/minute titrated to a 25% reduction in MAP.
B. Labetalol 2 mcg/minute titrated to a 50% reduction in MAP.
C. Sodium nitroprusside 0.25 mcg/kg/minute titrated to a 25% reduction in MAP.
D. Clonidine 0.1 mg orally every 2 hours as needed for a 50% reduction in MAP.
- Answer: A
Hypertensive emergency should be treated immediately
by no more than a 25% reduction in MAP over
the first hour, followed by a further reduction to a blood
pressure of 160/100 mm Hg over the next 2–6 hours.
The patient’s comorbidities guide the optimal therapy.
His dialysis and SCr of 11.4 mg/dL are contraindications
to sodium nitroprusside (Answer C is incorrect)
because of possible thiocyanate toxicity. Labetalol
(and β-blockers in general) is controversial in patients
who have taken cocaine, but its nonselective nature
makes it an option; however, this patient appears to
have decompensated HF, and a reduction of 50% initially
is too rapid a decrease in blood pressure for safety
(Answer B is incorrect). Clonidine is not appropriate
for a hypertensive emergency because its oral form is
difficult to titrate and can lead to precipitous drops in
blood pressure beyond the goal 25% reduction and possibly
stroke or worsening MI (Answer D is incorrect).
Nitroglycerin is optimal, considering the patient’s lack
of contraindications to this therapy and his evolving
MI and symptoms of HF (Answer A is correct).
- A 56-year-old white woman with a long history of hypertension because of nonadherence and recently diagnosed
HF (EF 35%) presents to the local ED with blood pressure 210/120 mm Hg and heart rate 105 beats/
minute. She states that she felt a little lightheaded but that she now feels okay. She ran out of her blood pressure
medications (including hydrochlorothiazide, carvedilol, and lisinopril) 3 days ago. Her current laboratory values
are within normal limits. Which medication is best for this patient?
A. Sodium nitroprusside 0.25 mcg/kg/minute titrated to a 25% reduction in MAP.
B. Labetalol 80 mg intravenously; repeat until blood pressure is less than 120/80 mm Hg.
C. Resumption of home medications; refer for follow-up within 2 days.
D. Resumption of home medications; initiate amlodipine 10 mg daily; refer for follow-up in 1 week.
- Answer: C
In an asymptomatic hypertensive crisis (without acute
target-organ damage), giving intravenous medications,
as in Answers A and B, and admitting the patient to
the hospital are unnecessary (Answers A and B are
incorrect). This patient is likely presenting because
of recent nonadherence. Resuming her home medications
(Answer C is correct) at this time would be most
appropriate, with a close follow-up to ensure that her
prescribed regimen is working. Adding a fourth agent
(Answer D is incorrect) at this time is unnecessary,
considering that her disease could be controlled on her
current drug regimen if she were adherent. Follow-up
should occur within the first few days, rather than waiting
1 week.
Questions 1 and 2 pertain to the following case.
G.G. is a 56-year-old white man with type 2 diabetes. He
is a nonsmoker and is concerned about his risk of coronary
heart disease (CHD). His vital signs include blood pressure
(BP) 152/98 mm Hg (average home blood pressure 150/92
mm Hg), heart rate 70 beats/minute, and body mass index
(BMI) 26.5 kg/m2. His fasting laboratory test results today
include serum creatinine (SCr) 0.8 mg/dL, total cholesterol
(TC) 188 mg/dL, low-density lipoprotein cholesterol
(LDL) 130 mg/dL, high-density lipoprotein cholesterol
(HDL) 30 mg/dL, and triglycerides (TG) 90 mg/dL, and his urine albumin/creatinine ratio is 86.5 mg/g (previously 68 mg/g). The patient’s 10-year atherosclerotic cardiovascular
disease (ASCVD) risk is 21%.
1. Which is most appropriate to recommend for this patient’s BP control at today’s visit?
A. Amlodipine 5 mg/day.
B. Lisinopril 10 mg/day.
C. Hydrochlorothiazide 12.5 mg/day plus amlodipine
5 mg/day.
D. Chlorthalidone 12.5 mg/day plus lisinopril 10 mg/day.
- Answer: D
The patient’s blood pressure is above his goal of less than
140/90 mm Hg (according to the ADA; can consider less
than 130/80 mm Hg because the 10-year ASCVD risk is
greater than 15%) and less than 130/80 mm Hg (according
to the ACC/AHA); thus, antihypertensive therapy should
be initiated. The patient has stage 2 HTN according to the
ACC/AHA guidelines; patients in this stage should be initiated
on two medications of different classes, particularly
if their blood pressure is more than 20/10 mm Hg above
goal; Answers A and B are incorrect because they only
recommend monotherapy. According to the 2021 ADA
standards, treatment of HTN should include drug classes
shown to reduce CV events in patients with diabetes (ACE
inhibitors, ARBs, thiazide-like diuretics, or dihydropyridine
CCBs), with the specific use of ACE inhibitors or
ARBs if the patient has albuminuria or CAD. The ACC/
AHA guidelines recommend ACE inhibitors and ARBs as
first-line treatment for patients with diabetes in the presence
of albuminuria, making Answer D correct. Calcium
channel blockers and thiazide diuretics are also recommended
as first-line agents. Answer D is correct because
it includes both a thiazide diuretic, which improved CV
outcomes in the ALLHAT trial, and an ACE inhibitor for
renal protection. Answer C is incorrect because it does
not include an ACE inhibitor or an ARB, one of which is
needed because the patient has albuminuria.
- Which is best for primary prevention of CHD for this patient?
A. Aspirin 325 mg/day.
B. Atorvastatin 10 mg/day.
C. Aspirin 81 mg/day plus atorvastatin 40 mg/day.
D. Atorvastatin 40 mg/day plus clopidogrel 75 mg/
day.
- Answer: C
Using the PCE risk calculator, this patient has an estimated
10-year ASCVD risk of 21%. Because the patient is 40–75
years old with diabetes and has an LDL of 70–189 mg/
dL, at least moderate-intensity statin therapy is indicated.
However, high-intensity statin therapy can be considered if
several risk factors exist or in patients 50–75 years of age,
according to the 2018 AHA/ACC Multisociety cholesterol
guidelines. Similarly, the ADA guidelines state that highintensity
statin therapy can be considered in patients with
diabetes with several ASCVD risk factors; this patient has
elevated blood pressure, dyslipidemia, and albuminuria
(Answer B is incorrect). For patients with diabetes, the
ADA recommends aspirin 75–162 mg for men 50 and older
who have at least one other risk factor for CHD (Answer
C is correct). If indicated, aspirin 81 mg can be used for
primary prevention if the patient is at low risk of bleeding
(Answer A is incorrect). Clopidogrel is recommended
for primary prevention only if the patient has an allergy to aspirin (Answer D is incorrect). Once the patient’s lipids
and blood pressure are better controlled, it may be appropriate
to discontinue aspirin therapy because the benefits
of therapy are closely balanced by risks, particularly in
those with a lower risk of ASCVD.
- J.R. is a 55-year-old Hispanic man (weight 66 kg)
with a medical history significant for HTN and a
myocardial infarction (MI) 9 months ago, leading to
placement of a bare metal stent (BMS). His current
medications include aspirin 325 mg/day, clopidogrel 75 mg/day, atorvastatin 40 mg/day, and metoprolol succinate 50 mg/day. His vital signs are BP 145/88 mm Hg and heart rate 52 beats/minute. His laboratory test results show LDL 68 mg/dL (baseline 138 mg/dL) and SCr 1.0 mg/dL. Which is currently best for secondary prevention of CHD?
A. Decrease the aspirin dose to 81 mg/day and add
lisinopril to 5 mg/day.
B. Increase atorvastatin to 80 mg/day and increase
metoprolol succinate to 100 mg/day.
C. Decrease the aspirin dose to 81 mg/day and
increase metoprolol succinate to 100 mg/day.
D. Discontinue clopidogrel and increase atorvastatin
to 80 mg/day.
- Answer: A
The patient’s heart rate is at less than 60 beats/minute.
Therefore, the metoprolol dose should not be increased
because of the risk of worsening bradycardia and heart
block, making Answers B and C incorrect. Because the
patient is taking a high-intensity statin with an adequate
response in his LDL (reduction of 50% or more from
baseline), his statin dose need not be increased, making
Answers B and D incorrect. The patient’s blood pressure is
not at goal (goal less than 130/80 mm Hg according to the
2017 AHA/ACC guidelines). An ACE inhibitor guideline
is a class I recommendation for secondary prevention in
patients with HTN. The aspirin dose after PCI associated
with the lowest bleeding risk is 81 mg, making Answer A
correct. Clopidogrel is indicated for at least 1 year after
PCI in the setting of ACS, making Answer D incorrect.